Buscar

Lista 5 - Respostas(1)

Faça como milhares de estudantes: teste grátis o Passei Direto

Esse e outros conteúdos desbloqueados

16 milhões de materiais de várias disciplinas

Impressão de materiais

Agora você pode testar o

Passei Direto grátis

Você também pode ser Premium ajudando estudantes

Faça como milhares de estudantes: teste grátis o Passei Direto

Esse e outros conteúdos desbloqueados

16 milhões de materiais de várias disciplinas

Impressão de materiais

Agora você pode testar o

Passei Direto grátis

Você também pode ser Premium ajudando estudantes

Prévia do material em texto

Fundamentos de Análise
5a Lista de Exercícios - 24/09/2020
Prof. Maurício Corrêa
Problema 1 Uma sequência (xn) diz-se periódica quando existe p 2 N tal que
xn+p = xn para todo n 2 N. Prove que toda sequência periódica convergente
é constante.
Resposta Seja (xn) uma sequência periódica e a = lim xn. Da definição de
sequência periódica, vemos que a subsequência (xp, x2p, x3p, . . . ) é constante
e, como tem limite igual a a, é uma subsequência constante, com todos
os termos iguais a a. Da mesma forma, conclui-se que as subsequências
(xk, xp+k, x2p+k, . . . ), para k = 1, 2, . . . , p � 1, são constantes. Mas todo
termo de (xn) pertente a alguma dessas subsequências, concluímos que (xn)
é constante. ⌅
Problema 2 Dadas as sequências (xn) e (yn), defina (zn) pondo z2n�1 = xn
e z2n = yn. Se lim xn = lim yn = a, prove que lim zn = a.
Resposta Pela definição de limite de sequências, dado qualquer " > 0, existe
n1 tal que n > n1 =) |xn � a| < " e n2 tal que n1 tal que n > n1 =)
|yn � a| < ". Defina n0 = max{2n1 � 1, 2n2}. Temos dois casos a considerar.
Se n = 2k�1, para algum k natural, então n > n0 =) 2k�1 > 2n1�1 =)
k > n1. Daí, usando a definição de zn e n1, temos, |zn � a| = |xk � a| < ".
No segundo caso, n = 2k. Então, n > n0 =) 2k > 2n2 =) k > n2.
Novamente, pela definição de zn, segue que |zn � a| = |yk � a| < ".
Portanto, em qualquer dos dois casos possíveis, n > n0 =) |zn� a| < ",
logo lim zn = a. ⌅.
Problema 3 Se lim xn = a, prove que lim |xn| = |a|.
Resposta Pela desigualdade triangular reversa ||xn| � |a||  |xn � a|. Já
que lim xn = a, para todo " > 0 existe um natural n0 tal que n > n0 =)
|xn � a| < ". Combinando esses dois fatos, vemos que, para todo " > 0,
existe n0 2 N tal que n > n0 =) ||xn|� |a||  |xn � a| < ", e lim |xn| = |a|.
⌅
Problema 4 Se lim xn = a, lim yn = b e |xn � yn| � " para todo n 2 N,
prove que |a� b| � "
1
Resposta Suponha |a � b| < ". Tome, então, "0 = " � |a � b| > 0. Então,
existem n1, n2 2 N tal que, para todo n > n1, |xn � a| < "0/2 e, para todo
n > n2, |yn � b| < "0/2. Tome n0 = max{n1, n2}. Daí, para todo n > n0,
|xn � yn|  |xn � a|+ |yn � b|+ |a� b|
<
"0
2
+
"0
2
+ |a� b|
= "0 + |a� b|
= ".
Portanto, se |xn � yn| � " para todo n, então |a� b| � ". ⌅
Problema 5 Sejam lim xn = a e lim yn = b. Se a < b, prove que existe
n0 2 N tal que n > n0 =) xn < yn.
Resposta Suponha que não existe n0 com tal propriedade. Daí, existe n1 2 N
com xn1 � yn1 . Pelo mesmo motivo, existe n2 > n1 tal que xn2 � yn2 . Prosse-
guindo dessa maneira, encontramos subsequências (x0k) = (xnk) e (y0k) = (ynk)
tais que xnk � ynk para todo k 2 N. Sendo (x0k) e (y0k) subsequências de (xn)
e (yn), temos que lim x0k = a e lim y0k = b. De x0k � y0k para todo k, segue que
a � b.
Então, se a < b, necessariamente existe n0 2 N tal que n > n0 =) xn <
yn. ⌅
Problema 6 Prove que, para todo p 2 N, limn!+1 n+p
p
n = 1.
Resposta Note que, para todo p natural,
1 < n+p
p
n < n
p
n.
Usando o fato que lim n
p
n = 1 e o Teorema do Sanduíche, temos que
limn!+1 n+p
p
n = 1. ⌅
Problema 7 Prove que lim n
p
n! = +1.
Resposta Sabemos, pelo exemplo 9 e o Teorema 9.3, que, para todo A > 1,
lim n!An = 1. Da definição de limite infinito, existe n0 2 N tal que n >
n0 =) n!An > 1 =) n! > A
n. Elevando os dois lados a 1/n, temos que,
para todo A > 1, existe n0 2 N tal que |n � n0| =) n
p
n! > A, isto é,
lim n
p
n! = +1. ⌅
2

Continue navegando